A geometry problem by Adrian Taboada

Geometry Level 5

A point P within an equilateral triangle has distance 4 cm, 5 cm, 6 cm, from the vertices. Find the length of the sides of the triangle.


The answer is 8.53.

This section requires Javascript.
You are seeing this because something didn't load right. We suggest you, (a) try refreshing the page, (b) enabling javascript if it is disabled on your browser and, finally, (c) loading the non-javascript version of this page . We're sorry about the hassle.

6 solutions

Bashing a little with law of cosines, we get the following formula:

L = a 2 + b 2 + c 2 + 3 ( a + b + c ) ( a + b c ) ( a b + c ) ( a + b + c ) 2 L=\sqrt{\dfrac{a^2+b^2+c^2+\sqrt{3(a+b+c)(a+b-c)(a-b+c)(-a+b+c)}}{2}}

where L L is the side of the equilateral triangle and a a , b b and c c are the distances from P P to the vertices of the triangle.

Substituting we get:

L = 4 2 + 5 2 + 6 2 + 3 ( 4 + 5 + 6 ) ( 4 + 5 6 ) ( 4 5 + 6 ) ( 4 + 5 + 6 ) 2 L=\sqrt{\dfrac{4^2+5^2+6^2+\sqrt{3(4+5+6)(4+5-6)(4-5+6)(-4+5+6)}}{2}}

L = 77 + 4725 2 L=\sqrt{\dfrac{77+\sqrt{4725}}{2}}

L = 77 + 15 21 2 L=\sqrt{\dfrac{77+15\sqrt{21}}{2}}

L 8.5363 L \approx 8.5363

how did you derive the formulae

abhishek alva - 4 years, 11 months ago

Let the equilateral triangle Δ A B C \Delta ABC be such that A B = B C = A C = x , A P = 4 , B P = 5 AB = BC = AC = x, AP = 4, BP = 5 and B C = 6 BC = 6 .

Now let A P B = α \angle APB = \alpha and A P C = β \angle APC = \beta . Then B P C = 2 π ( α + β ) \angle BPC = 2\pi - (\alpha + \beta) . Now use the Cosine Law on triangles Δ A P B , Δ A P C \Delta APB, \Delta APC and Δ B P C \Delta BPC to establish the respective equations

(i) x 2 = 41 40 cos ( α ) x^{2} = 41 - 40\cos(\alpha) ,

(ii) x 2 = 52 48 cos ( β ) x^{2} = 52 - 48\cos(\beta) , and

(iii) x 2 = 61 60 cos ( α + β ) . x^{2} = 61 - 60\cos(\alpha + \beta).

This system has (second quadrant) solution values α = 2.49267 , β = 2.02059 \alpha = 2.49267, \beta = 2.02059 radians. Plugging this value of α \alpha into (i) yields the solution x = 8.536 x = \boxed{8.536} to 3 3 decimal places.

(Note that the same value for x x is found when this value for β \beta is plugged into (ii).)

Can you please explain the steps on how you solved the system for the values of α and β?

Deepkaran Ghumman - 6 years, 6 months ago

Log in to reply

I actually just relied on WolframAlpha, since the algebra is really messy. However, I will sketch out an outline of a method to find x x directly.

Let γ = 2 π ( α + β ) \gamma = 2\pi - (\alpha + \beta) . Then

cos ( γ ) = cos ( α + β ) = cos ( α ) cos ( β ) sin ( α ) sin ( β ) \cos(\gamma) = \cos(\alpha + \beta) = \cos(\alpha)\cos(\beta) - \sin(\alpha)\sin(\beta) \Longrightarrow

( cos ( α ) cos ( β ) cos ( γ ) ) 2 = sin 2 ( α ) sin 2 ( β ) = ( 1 cos 2 ( α ) ) ( 1 cos 2 ( β ) ) . (\cos(\alpha)\cos(\beta) - \cos(\gamma))^{2} = \sin^{2}(\alpha)\sin^{2}(\beta) = (1 - \cos^{2}(\alpha))(1 - \cos^{2}(\beta)).

Call this last equation (A). Now from (i) through (iii) we have that

cos ( α ) = 41 x 2 40 , cos ( β ) = 52 x 2 48 , cos ( γ ) = 62 x 2 60 . \cos(\alpha) = \dfrac{41 - x^{2}}{40}, \cos(\beta) = \dfrac{52 - x^{2}}{48}, \cos(\gamma) = \dfrac{62 - x^{2}}{60}.

We can then substitute these expressions into equation (A) and rearrange to get a quartic in x x . (This is the really messy part, so I'll skip to the punch line.) This quartic turns out to be

x 4 77 x 2 + 301 = 0 x 2 = 77 ± 15 21 2 x = 1 2 154 ± 30 21 x^{4} - 77x^{2} + 301 = 0 \Longrightarrow x^{2} = \dfrac{77 \pm 15\sqrt{21}}{2} \Longrightarrow x = \dfrac{1}{2} \sqrt{154 \pm 30\sqrt{21}} .

The smaller of the two possible values can be discarded as the distances from the vertices to P P would leave P P outside the triangle, so we are left with the solution x = 1 2 154 + 30 21 = 8.536 x = \frac{1}{2} \sqrt{154 + 30\sqrt{21}} = 8.536 as found before, (but at least now we have an exact value for x x ).

Brian Charlesworth - 6 years, 6 months ago

i wrote the cosine formula for each angle and then i also applied lami's theorm here(its in physics, but, thought i would give it a try here), considering that 4,5,6 are the magnitudes of the forces. i got the answer 8.72. this method's pretty viable, yes? but i am not getting the exact answer..

Lipsa Kar - 5 years, 3 months ago

Log in to reply

That's an interesting idea, but I don't see how there is a physical correlation between the lengths involved in this question with forces as dealt with in Lami's theorem. Also, when I combined my cosine formulas with the Lami equations I didn't end up with real solutions.

Brian Charlesworth - 5 years, 3 months ago

Log in to reply

i assumed they are the forces' magnitude. so, then the method can be applied,no? i got the value of the side length as root of 76.

Lipsa Kar - 5 years, 3 months ago

Log in to reply

@Lipsa Kar I don't think that we can consider the lengths given as being "equivalent" to forces, so Lami's theorem cannot be applied in this case.

Brian Charlesworth - 5 years, 3 months ago

For equation 3, wouldn't it be cos(2pi-(a+b))? Sorry for the lack of Latex.

Rohan Khajuria - 5 years, 11 months ago

Log in to reply

Yes, but I simplified it since cos ( 2 π ( α + β ) ) = cos ( α + β ) . \cos(2\pi - (\alpha + \beta)) = \cos(\alpha + \beta).

Brian Charlesworth - 5 years, 11 months ago
Oliver Piattella
Jan 16, 2016

Put the equilateral triangle A B C \triangle ABC in a Cartesian plane, with one of the vertices say C coinciding with the origin. Therefore, A = ( l / 2 , 3 l / 2 ) A=(l/2,\sqrt{3}l/2) and B = ( l , 0 ) B = (l, 0) , where l l is the length of the side of the triangle. Let P have coordinates ( x , y ) (x, y) .

Now, compute the distances of the vertices to P. I assume C P = 6 CP = 6 , A P = 5 AP = 5 and B P = 4 BP = 4 . One has: x 2 + y 2 = 36 , ( l x ) 2 + y 2 = 16 , ( x l / 2 ) 2 + ( 3 l / 2 y ) 2 = 25 . x^2 + y^2 = 36\;,\\ (l - x)^2 + y^2 = 16\;,\\ (x - l/2)^2 + (\sqrt{3}l/2 - y)^2 = 25\;.

Manipulating this system of equations, one arrives at the following biquadratic equation: l 4 77 l 2 + 301 = 0 . l^4 - 77l^2 + 301 = 0\;.

The only viable solution is l = 15 21 2 + 77 2 8.53635 l = \sqrt{\frac{15 \sqrt{21}}{2}+\frac{77}{2}} \approx \boxed{8.53635} . The other positive solution is l = 2.03241 l = 2.03241 but it is unacceptable, since it would imply P to be outside the triangle.

Moderator note:

This (slightly) brute force approach works. Unfortunately, there isn't much nice aspects of this problem that could be exploited.

Saarthak Marathe
Apr 17, 2016

The relation,

3 ( p 4 + q 4 + r 4 + a 4 ) = ( p 2 + q 2 + r 2 + a 2 ) 2 3({p}^{4}+{q}^{4}+{r}^{4}+{a}^{4})={({p}^{2}+{q}^{2}+{r}^{2}+{a}^{2})}^{2} is always true for an equilateral triangle.

Where, p , q , r p,q,r are the distances of the point from the vertices, a a is the length of the side of the triangle.

This relation can be proved using co-ordinate geometry and some bashing.

After this we can substitute the values and get the answer as 8.5364 8.5364 .

@Saarthak Marathe What is the name of this relation ? Or, can you write the proof here?

Dexter Woo Teng Koon - 4 years, 4 months ago
Tapas Mazumdar
Oct 3, 2016

I have applied coordinate geometry to solve this problem.


Consider the equilateral shown above in the Cartesian plane with side length a a units with vertices at ( 0 , 0 ) (0,0) , ( a 2 3 a 2 ) \left(\dfrac a2 \, \dfrac{\sqrt 3 a}{2}\right) and ( a , 0 ) (a,0) . Let coordinates of position of P P be ( x , y ) (x,y) with distances 5 5 , 4 4 and 6 6 units respectively from the three vertices.

Using distance formula, we get,

( x a 2 ) 2 + ( y 3 a 2 ) 2 = 16 ( 1 ) x 2 + y 2 = 25 ( 2 ) ( x a ) 2 + y 2 = 36 ( 3 ) {\left( x - \dfrac a2 \right)}^2 + {\left( y - \dfrac{\sqrt3 a}{2} \right)}^2 = 16 \ \cdots (1) \\ x^2 + y^2 = 25 \ \cdots (2) \\ {(x-a)}^2 + y^2 = 36 \ \cdots (3)

Expanding ( 1 ) (1) , we get,

x 2 + a 2 4 a x + y 2 + 3 a 2 4 3 a y = 16 ( 4 ) x^2 + \dfrac{a^2}{4} - ax + y^2 + \dfrac{3 a^2}{4} - \sqrt{3} ay = 16 \ \cdots (4)

Expanding ( 3 ) (3) , we get,

x 2 + a 2 2 a x + y 2 = 36 ( 5 ) x^2 + a^2 -2ax + y^2 = 36 \ \cdots (5)

Substituting x 2 + y 2 x^2 + y^2 with 25 25 [from ( 2 ) (2) ] in ( 4 ) (4) and ( 5 ) (5) , we get,

25 + a 2 a x 3 a y = 16 a 2 a x 3 a y = 9 ( 6 ) 25 + a 2 2 a x = 36 a 2 2 a x = 11 ( 7 ) 25 + a^2 - ax - \sqrt{3} ay = 16 \implies a^2 - ax - \sqrt{3} ay = -9 \ \cdots (6) \\ 25 + a^2 -2ax = 36 \implies a^2 -2ax = 11 \ \cdots (7)

Using ( 6 ) (6) and ( 7 ) (7) , we obtain the value of x x and y y in terms of a a as:

x = a 2 11 2 a y = a 2 + 29 2 3 a \color{#20A900}{x = \dfrac{a^2 - 11}{2a} \\ y = \dfrac{a^2 + 29}{2 \sqrt3 a}}

Plugging in the values of x x and y y in ( 2 ) (2) , we obtain,

( a 2 11 2 a ) 2 + ( a 2 + 29 2 3 a ) 2 = 25 {\left( \dfrac{a^2 - 11}{2a} \right)}^2 + {\left( \dfrac{a^2 + 29}{2 \sqrt3 a} \right)}^2 = 25

a 4 77 a 2 + 301 = 0 \implies a^4 - 77 a^2 + 301 = 0

Whose, positive roots (as we are required to find magnitude of length, so we are considering only positive roots) are given as:

a = 77 + 15 21 2 or a = 77 15 21 2 a = \sqrt{\dfrac{77 + 15 \sqrt{21}}{2}} \ \text{or} \ a = \sqrt{\dfrac{77 - 15 \sqrt{21}}{2}}

The value of 77 15 21 2 \sqrt{\dfrac{77 - 15 \sqrt{21}}{2}} comes out to be approximately 2.03 2.03 , which implies that P P lies outside our triangle as each of the distances from P P to each of the vertices is greater than 2.03 2.03 .

But, since, P P lies within our triangle, so we have,

a = 77 + 15 21 2 8.53 a = \sqrt{\dfrac{77 + 15 \sqrt{21}}{2}} \approx \boxed{8.53}

i had to use wolfram alpha you can see it here : " https://www.wolframalpha.com/input/?i=sqrt(x%5E2-1) sqrt(81-x%5E2)%2B+sqrt(x%5E2-4) sqrt(100-x%5E2)%2Bsqrt(x%5E2-1) sqrt(121-x%5E2)-sqrt+(3) x%5E2%3D0 " i raeched these equations using the area constraint.

0 pending reports

×

Problem Loading...

Note Loading...

Set Loading...